FAR Section 3- Select Transactions

Ace your homework & exams now with Quizwiz!

Each of the following would be considered a Level 2 observable input that could be used to determine an asset or liability's fair value, except: quoted prices for identical assets and liabilities in markets that are not active. quoted prices for similar assets and liabilities in markets that are active. internally generated cash flow projections for a related asset or liability. interest rates that are observable at commonly quoted intervals.

internally generated cash flow projections for a related asset or liability. Based on the following definition, "internally generated cash flow projections for a related asset or liability" is the only answer choice not included in Level 2 observable inputs. "If the asset or liability has a specified (contractual) term, a Level 2 input must be observable for substantially the full term of the asset or liability. Level 2 inputs include the following: "Quoted prices for similar assets or liabilities in active markets "Quoted prices for identical or similar assets or liabilities in markets that are not active "Inputs other than quoted prices that are observable for the asset or liability, for example: "Interest rates and yield curves observable at commonly quoted intervals "Implied volatilities "Subparagraph superseded by Accounting Standards Update No. 2011-04 "Subparagraph superseded by Accounting Standards Update No. 2011-04 "Credit spreads "Subparagraph superseded by Accounting Standards Update No. 2011-04 "Market-corroborated inputs."

On December 31, Year 6, Bradley Corp. sold a piece of land with a carrying cost of $1 million to Kravis Inc. for $2 million and simultaneously leased it back for 10 years with annual payments of $120,000 payable at the end of each year using a 5% discount rate. The lease is appropriately recorded as a sale and leaseback transaction by both Bradley and Kravis. The market rates for the lease of the land are $90,000, payable annually at the end of each year. On Bradley's December 31, Year 6, balance sheet, the recognized gain from the sale of land would be: $0. $8,200. $60,000. $68,200.

$0 Since the transaction is recorded as a sale/leaseback, the gain is recognized at the lease's commencement on the income statement, not the balance sheet.

Milt Co. began operations on January 1, Year 1. On January 1, Year 3, Milt changed its inventory method from LIFO to FIFO for both financial and income tax reporting. If FIFO had been used in prior years, Milt's inventories would have been higher by $60,000 and $40,000 at December 31, Year 3 and Year 2, respectively. Milt has a 30% income tax rate. What amount should Milt report as the cumulative effect of this accounting change in its income statement for the year ended December 31, Year 3? $0 $14,000 $28,000 $42,000

$0 The cumulative effect of a change in an accounting method is reflected in retained earnings, not net income.

On September 1, 20X1, Brady Corp. entered into a foreign exchange contract for speculative purposes by purchasing 50,000 deutsche marks for delivery in 60 days. The rates to exchange $1 for 1 deutsche mark follow: 09/01/X1 09/30/X1 Spot rate .75 .70 30-day forward rate .73 .72 60-day forward rate .74 .73 In its September 30, 20X1, income statement, what amount should Brady report as foreign exchange loss? $2,500 $1,500 $1,000 $500

$1,000 Foreign currency amount at 09/01/X1 ($.74 x 50,000) $37,000 Less: Foreign currency amount at 09/30/X1 ($.72 x 50,000) 36,000 Foreign exchange loss $ 1,000 ======= Note: On September 1, 20X1, the 60-day forward rate is used while the rate used on September 30, 20X1, is the 30-day rate. Since the forward contract (the derivative) is for purposes of speculation, any associated exchange gain or loss must be included in net income. FASB ASC 815-25-35-15 addresses the use of hedges to control for foreign currency exposure. If a derivative qualifies as a hedge, FASB ASC 815-25-35-1 permits companies to match the timing of the gains and losses of hedged items and their hedging derivatives. For a fair value hedge, FASB ASC 815-25-35-1 permits the hedger to record the change in the fair value of the hedged item concurrently with the gain or loss on the hedging derivative. For a cash flow hedge, the effective portion of any changes in the hedging derivative's fair value is recorded in other comprehensive income until the change in the value of the hedged item is recognized in earnings. If a derivative does not qualify as a hedge, changes in its value must be reported in quarterly earnings. Speculative contracts do not quality as hedges and are still controlled by FASB ASC 830-20-35-1. For these derivatives, the change in fair value is recognized immediately in net income.

Troy Corporation purchased merchandise for 25,000 Australian dollars from a vendor in Melbourne on December 1, 20X4. Payment in Australian dollars was due on January 31, 20X5. The exchange rates to purchase one Australian dollar were as follows: 12/1/X4 12/31/X4 Spot-rate $0.74 $0.78 30-day rate 0.75 0.79 60-day rate 0.77 0.80 In its December 31, 20X4, income statement, what amount should Troy report as the effect of this foreign exchange transaction? $250 gain $1,000 gain $250 loss $1,000 loss

$1,000 loss FASB ASC 830-20 (Foreign Currency Transactions) provides that a gain or loss on a forward contract is computed by multiplying the foreign currency amount of the forward contract by the difference between the spot rate at the balance sheet date and the spot rate at the date of inception of the forward contract. Troy's reported foreign exchange loss: = 25,000 Australian dollars × ($0.74 - $0.78) per Australian dollar = $1,000 loss

The following information pertains to Stone Corporation's sale of 45,000 foreign currency units under a forward contract dated April 1, 20X4, for delivery on July 31, 20X4: 4/01/X4 6/30/X4 Spot rates $1.13 $1.09 30-day future rates 1.15 1.13 90-day future rates 1.16 1.15 Stone has a fiscal year end of June 30. The reason Stone entered into the forward contract is to speculate and earn a return. In Stone's income statement for the year ended June 30, 20X4, what effect should be reported from this forward contract? $1,800 loss $1,350 loss $1,350 gain $1,800 gain

$1,350 gain Forward rate available for remaining maturity of contract (30-day rate at 6/30/X4) $ 1.13 Less contracted forward rate (90-day future rate at 4/1/X4) 1.16 Difference $ (.03) Multiplied by foreign currency units x 45,000 Gain on forward contract to be reported in 20X4 $ 1,350 ======== If a derivative qualifies as a hedge, FASB ASC 815-25-35-1 permits companies to match the timing of the gains and losses of hedged items and their hedging derivatives. For a fair value hedge, the hedger can record the change in the fair value of the hedged item concurrently with the gain or loss on the hedging derivative. For a cash flow hedge, the effective portion of any changes in the hedging derivative's fair value is recorded in other comprehensive income until the change in the value of the hedged item is recognized in earnings. If a derivative does not qualify as a hedge, changes in its value must be reported in quarterly earnings.

On December 10 of the current year, a U.S. company sold merchandise on account to a Brazilian company for 84,000 real (exchange rate: 1 real = $0.27). At the company's December 31 fiscal year-end, the exchange rate was 1 real = $0.26. The exchange rate was 1 real = $0.24 on collection in early February of the subsequent year. What amount would the company recognize as a gain or loss from this foreign currency transaction when the receivable is collected? $1,680 gain $840 gain $1,680 loss $840 loss

$1,680 loss The company would recognize a $1,680 loss (note that the account would have been adjusted at December 31 to $21,840): January (84,000 real × $0.24) $20,160 December (84,000 real × $0.26) 21,840 Loss $1,680

Wall Co. leased office premises to Fox, Inc., for a 5-year term beginning January 2, 20X1. Under the terms of the operating lease, rent for the first year is $8,000 and rent for Years 2 through 5 is $12,500 per annum. However, as an inducement to enter the lease, Wall granted Fox the first six months of the lease rent-free. In its December 31, 20X1, income statement, what amount should Wall report as rental income? $12,000 $11,600 $10,800 $8,000

$10,800 Rent for Year 1 $ 8,000 x 6/12 = $ 4,000 Rent for Years 2-5 $12,500 x 4 = 50,000 Total for five years $54,000 ======= Rent for 1 year (amount to be reported for 20X1) = $54,000 / 5 = $10,800

On January 2, Year 4, Raft Corp. discovered that it had incorrectly expensed a $210,000 machine purchased on January 2, Year 1. Raft estimated the machine's original useful life to be 10 years and its salvage value at $10,000. Raft uses the straight-line method of depreciation and is subject to a 30% tax rate. In its December 31, Year 4, financial statements, what amount should Raft report as an adjustment to the opening balance of retained earnings of the earliest year presented? (Assume that only one year is presented.) $102,900 $105,000 $165,900 $168,000

$105,000 A correction of an error in prior years' financial statements is reported in the year of correction by restating all prior years affected by the error. The cumulative effect of the error on periods prior to those presented must be reflected in the carrying amounts of the assets and liabilities as of the beginning of the earliest year presented in the current period's financial report. In addition, the offsetting amount of this cumulative effect must be reported as an adjustment to the opening balance of retained earnings of the earliest year presented in the current period's financial report. (FASB ASC 250-10-45-23 to 45-24) Depreciation of $20,000 should have been recognized in Years 1-4, as well as each of the remaining six years of the asset's life after Year 4. However, no depreciation was recorded in any of the years up to Year 4. Rather, an expense of $210,000 was recognized in Year 1. Correct Expensed Cumulative Cumulative Depreciation Depreciation (Before Tax) (After Tax) ------------ ------------ ------------ ----------- Year 1 $20,000 $210,000 $190,000 $133,000 Year 2 20,000 0 170,000 119,000 Year 3 20,000 0 150,000 105,000

On February 3, Rafia Corporation, a U.S. company, sold and delivered merchandise on a 30-day account to Sanchez & Company, a Mexican corporation, for 1,900,000 pesos. On March 3, Sanchez paid Rafia in full. Relevant currency exchange rates were as follows: February 3 March 3 Spot rate $ .056 $ .050 30-day forward rate .052 .049 What amount should Rafia record to accounts receivable when it receives payment from Sanchez on March 3 as an account receivable for its sale to Sanchez? $106,400 $95,000 $93,100 $11,400

$106,400 The receivable should initially be recorded at the spot rate on the date of the transaction (1,900,000 pesos × $0.056 = $106,400). When Rafia receives payment on March 3, the 1,900,000 pesos will equal $95,000 (1,900,000 × $0.05) in U.S. dollars, not the $106,400 U.S. dollars for which the accounts receivable was originally recorded. The journal entry on March 3 is: Cash 95,000 Foreign currency transaction loss 11,400 Accounts receivable 106,400

Cody Corporation planned to depreciate a $383,000 asset over seven years, using the straight-line method with a salvage value of $19,000. At the beginning of the third year, it was determined that the asset would only last two more years. What amount should Cody report as the asset's carrying value at the end of year 3, after depreciation for the year had been recorded? $279,000 $260,000 $149,000 $130,000

$149,000 Cody should report $2,800 depreciation expense for the year ending December 31, year 6: Original cost $383,000 Salvage value (19,000) Depreciable basis 364,000 Depreciation Years 1-3 ($52,000 x 2 yrs.) 104,000 Depreciable basis at beginning of Year 3 260,000 Year 3 depreciation ($260,000 / 2) 130,000 Carrying value at end of year 3 ($260,000 - $130,000) + $19,000 = $149,000

On December 30, 20X1, Ames Co. leased equipment under a finance lease for 10 years. It contracted to pay $40,000 annual rent on December 31, 20X1, and on December 31 of each of the next nine years. The finance lease liability was recorded at $270,000 on December 30, 20X1, before the first payment. The equipment's useful life is 12 years, and the interest rate implicit in the lease is 10%. Ames uses the straight-line method to depreciate all equipment. In recording the December 31, 20X2, payment, by what amount should Ames reduce the finance lease liability? $27,000 $23,000 $22,500 $17,000

$17,000 Finance lease liability before first payment $270,000 Less December 31, 20X1, payment (all attributed to reduction in lease liability) 40,000 Finance lease liability at 01/01/X2 $230,000 ======== December 31, 20X2, lease payment $40,000 Less interest portion (10% x 230,000) 23,000 Amount considered reduction in lease liability $17,000 =======

Winn Co. manufactures equipment that is sold or leased. On December 31, Year 1, Winn leased equipment to Bart for a 5-year period ending December 31, Year 6, at which date ownership of the leased asset will be transferred to Bart. Equal payments under the lease are $22,000 (including $2,000 executory costs) and are due on December 31 of each year. The first payment was made on December 31, Year 1. Collectibility of the remaining lease payments is reasonably assured, and Winn has no material cost uncertainties. The normal sales price of the equipment is $77,000, and cost is $60,000. On December 31, Year 1, what amount of income should Winn realize from the lease transaction? $17,000 $22,000 $23,000 $33,000

$17,000 When accounting for a lease as a sales-type or direct financing lease for a lessor, one of the lease criteria must be met. In this situation, one criterion is a transfer of title at the end of the lease. Also, when accounting for one of these leases from the perspective of the lessor, one must further decide if the lease is sales-type or direct financing. Since the transfer of title criterion is met, this is considered a sales-type lease and there could be an element of gross profit recognized by the lessor at the beginning of the lease, generally based on the difference between the sales price and the cost of the item to the lessor. Here that gross profit recognized immediately is $17,000, the difference between the selling price and the cost of the equipment ($77,000 - $60,000).

On November 2, 20X1, Platt Co. entered into a 90-day futures contract to purchase 50,000 Swiss francs when the contract quote was $0.70. The purchase was for speculation in price movement. The following exchange rates existed during the contract period: 30-Day Futures Spot Rate November 2, 20X1 $.62 $.63 December 31, 20X1 .65 .64 January 30, 20X2 .65 .68 What amount should Platt report as foreign currency exchange loss in its income statement for the year ended December 31, 20X1? $2,500 $3,000 $3,500 $4,000

$2,500 Futures contracts are a selected type of derivative instrument. All derivatives must be recognized on the balance sheet at fair value. Fair value is $0.70 on November 2, 20X1. Accounting for the changes in fair value depends on whether it has been designated as and qualifies for hedge accounting. Platt Co. has not hedged the risk of the futures contract and FASB ASC 815-20-35-1 specifies that gains and losses must be included in income for these contracts. Since this is a futures contract, the future 30-day rate ($0.65) is used to measure the gain or loss for the year ended December 31, 20X1. The foreign currency exchange loss for 20X1 is ($.70 - $.65) × 50,000 = $2,500.

Brand Co. incurred the following research and development project costs at the beginning of the current year: Equipment purchased for current and future projects $100,000 Equipment purchased for current projects only 200,000 Research and development salaries for current project 400,000 Equipment has a 5-year life and is depreciated using the straight-line method. What amount should Brand record as depreciation for research and development projects at December 31? $0 $20,000 $60,000 $140,000

$20,000 Only the equipment that has an alternative use is capitalized and depreciated. The depreciation is $100,000 ÷ 5 = $20,000. The other expenditures should be expensed immediately. FASB ASC 730-10-25-2 states: "Elements of costs shall be identified with research and development activities as follows (see [FASB ASC] 350-50 for guidance related to website development): "Materials, equipment, and facilities. The costs of materials (whether from the entity's normal inventory or acquired specially for research and development activities) and equipment or facilities that are acquired or constructed for research and development activities and that have alternative future uses (in research and development projects or otherwise) shall be capitalized as tangible assets when acquired or constructed. The cost of such materials consumed in research and development activities and the depreciation of such equipment or facilities used in those activities are research and development costs. However, the costs of materials, equipment, or facilities that are acquired or constructed for a particular research and development project and that have no alternative future uses (in other research and development projects or otherwise) and therefore no separate economic values are research and development costs at the time the costs are incurred."

On November 1, 20X1, Davis Co. discounted with recourse at 10% a 1-year, noninterest bearing, $20,500 note receivable maturing on January 31, 20X2. What amount of contingent liability for this note must Davis disclose in its financial statements for the year ending December 31, 20X1? $0 $20,000 $20,333 $20,500

$20,500 "With recourse" means that the financial entity where the note was discounted can collect fully from Davis Co. in the event the maker does not honor the note. Davis Co. should disclose the full amount ($20,500) of the note as a contingent liability in its December 31, 20X1, financial statements, because it is reasonably possible that Davis will have to make good on the note (FASB ASC 450-20-50-2).

Jarvis Corporation acquired a financial asset in the early part of 20X9. Jarvis needs to report the fair value of this asset in its 20X9 balance sheet. There is no principal market for the financial asset. The asset is actively traded on two different exchanges (Market 1 and Market 2). The information from these two exchanges is as follows: Quoted Price of Asset Transaction Costs Market 1 $24,000 $ 225 Market 2 23,950 140 What is the fair value of the financial asset? $24,000 $23,950 $23,810 $23,775

$23,950 Under FASB ASC 820-10-35-5, if there is no principal market, the entity should use the most advantageous market for that asset. FASB ASC 820-10-20 defines the most advantageous market as "the market that maximizes the amount that would be received to sell the asset or minimizes the amount that would be paid to transfer the liability, after taking into account transaction costs and transportation costs." Considering the transaction prices: Market 1 would result in $24,000 - $225 = $23,775 Market 2 would result in $23,950 - $140 = $23,810 Therefore, Market 2 would be the most advantageous market. Once the most advantageous market is identified, the quoted price of the asset is used, ignoring the transaction costs; therefore, the asset would be valued at $23,950.

Ball Corp. had the following foreign currency transactions during the current year: Goods purchased from a foreign supplier on January 20 for the U.S. dollar equivalent of $90,000. The invoice was paid on March 20, at the U.S. dollar equivalent of $96,000. On July 1, Ball borrowed the U.S. dollar equivalent of $500,000 evidenced by a note that was payable in the lender's local currency on July 1, in two years. On December 31, the U.S. dollar equivalents of the principal amount and accrued interest were $520,000 and $26,000, respectively. Interest on the note is 10% per annum. In Ball's year-end income statement, what amount should be included as foreign exchange loss? $0 $6,000 $21,000 $27,000

$27,000 Ball's year-end income statement should include $27,000 as foreign exchange loss, calculated as follows: Foreign currency loss on goods purchased ($90,000 - $96,000) $ 6,000 Loan principal foreign currency loss ($500,000 - $520,000) 20,000 Loan interest foreign currency loss ($25,000* - $26,000) 1,000 Total loss $27,000 * Interest in U.S. dollars: $500,000 × 0.10 × 1/2 year = $25,000

Miller Co. incurred the following computer software costs for the development and sale of software programs during the current year: Planning costs $ 50,000 Design of the software 150,000 Substantial testing of the project's initial stages 75,000 Production and packaging costs for the first month's sales 500,000 Costs of producing product masters after technology feasibility was established 200,000 The project was not under any contractual arrangement when these expenditures were incurred. What amount should Miller report as research and development expense for the current year? $200,000 $275,000 $500,000 $975,000

$275,000 FASB ASC 730-10-20 defines research and development as the planned search and critical investigation aimed at the discovery of new knowledge and ultimately a new product. Computer software costs follow this definition. Computer software costs to be sold, leased, or otherwise marketed are charged to expense as research and development until technological feasibility has been established for the product. After feasibility has been established, all software costs are capitalized. Based on the definition, research and development expense includes planning, design and testing of $275,000 ($50,000 + 150,000 + 75,000).

On January 1, year 1, a company with a calendar year-end began developing a software program that it intends to market and sell to its customers. The software coding was completed on March 31, year 1, at a cost of $200,000, and the software testing was completed on June 30, year 1, at a cost of $100,000. The company achieved technological feasibility on July 31, year 1, at which time the company began producing product masters at a cost of $125,000. What amount should the company report for the total research and development expense for the year ended December 31, year 1? $100,000 $200,000 $300,000 $425,000

$300,000 Computer software costs to be sold, leased, or otherwise marketed are charged to expense as research and development until technological feasibility has been established for the product. Technological feasibility is established on completion of a detailed program design or completion of a working model. After technological feasibility has been established, all software production costs are capitalized and subsequently reported at the lower of unamortized cost or net realizable value. The company should report $300,000 ($200,000 + $100,000) for research and development expense.

Nect Corporation reported a retained earnings balance of $330,000 at December 31 of the previous year. In August of the current year, Nect determined that insurance premiums of $51,000 for the 2-year period beginning January 1 of the previous year had been paid and fully expensed in that year. Nect has a 40% income tax rate. What amount should Nect report as adjusted beginning retained earnings in its current-year statement of retained earnings? $330,000 $345,300 $360,600 $319,800

$345,300 The beginning retained earnings balance should reflect the after-tax effect of only half of the insurance premium because the premium covered two years. Nect should report $345,300: Beginning retained earnings as originally reported $330,000 Overstated expense of $51,000/2 - Tax of 40% ($10,200) 15,300 Corrected beginning retained earnings $345,300

During 20X8, Detrusions Corporation incurred research costs of $178,000 and development costs of $256,000 in creating a new process for taking biopsies for cancer testing. The patent was granted on June 21, 20X8, and the patent was officially registered on July 1, 20X8. The costs to register the patent equaled $45,500. The patent's legal life is 20 years, and its estimated economic life is 7 years. In its December 31, 20X9, balance sheet, what amount should Detrusions report as patent, net of accumulated amortization, assuming no impairment? $45,500 $42,250 $39,000 $35,750

$35,750 FASB ASC 730-10-25-1 provides that research and development costs be "charged to expense when incurred." The $45,500 cost of registration would be capitalized and amortized over the 7-year economic life. 20X8 amortization = ($45,500 / 7 years) × (6/12) = $3,250 20X9 amortization = ($45,500 / 7 years) = $6,500 Carrying value of patent on December 31, 20X9 = Costs of patent − Patent amortization = $45,500 − $3,250 − $6,500 = $35,750

On April 1, Year 1, Hall Fitness Center leased its gym to Dunn Fitness Center under a 4-year operating lease. Hall normally charges $6,000 per month to lease its gym, but as an incentive, Hall gave Dunn half off the first year's rent, and one-quarter off the second year's rent. Dunn's rental payments were as follows: Year 1 12 x $3,000 = $36,000 Year 2 12 x $4,500 = $54,000 Year 3 12 x $6,000 = $72,000 Year 4 12 x $6,000 = $72,000 Dunn's rent payments were due on the first day of the month, beginning on April 1, Year 1. What amount should Dunn report as rent expense in its monthly income statement for April, Year 3? $6,000 $4,500 $4,875 $3,000

$4,875 Non-level lease payments must be expensed on a straight-line basis unless another systematic method is more representative. Year 1 12 x $3,000 = $ 36,000 Year 2 12 x $4,500 = 54,000 Year 3 12 x $6,000 = 72,000 Year 4 12 x $6,000 = 72,000 Total rent payments $ 234,000 Lease term / 48 Monthly rent $ 4,875

The following information pertains to Flint Co.'s sale of 10,000 foreign currency units under a forward contract dated November 1, 20X1, for delivery on January 31, 20X2: 11/01/X1 12/31/X1 Spot rates $0.80 $0.83 30-day future rates 0.79 0.82 90-day future rates 0.78 0.81 Flint entered into the forward contract in order to speculate in the foreign currency. In Flint's income statement for the year ended December 31, 20X1, what amount of loss should be reported from this forward contract? $300 $400 $0 $200

$400 Forward rate available for remaining maturity of contract (30-day rate at 12/31/X1) $ .82 Less contracted forward rate (90-day future rate at 11/01/X1) .78 Difference $ .04 Multiplied by foreign currency units x 10,000 Loss on forward contract to be reported in 20X1 $ 400 ======= FASB ASC 815-25-35-15 addresses the use of hedges to control for foreign currency exposure. If a derivative qualifies as a hedge, FASB ASC 815-25-35-1 permits companies to match the timing of the gains and losses of hedged items and their hedging derivatives. For a fair value hedge, FASB ASC 815-25-35-1 permits the hedger to record the change in the fair value of the hedged item concurrently with the gain or loss on the hedging derivative. For a cash flow hedge, the effective portion of any changes in the hedging derivative's fair value is recorded in other comprehensive income until the change in the value of the hedged item is recognized in earnings. If a derivative does not qualify as a hedge, changes in its value must be reported in quarterly earnings. Speculative contracts do not quality as hedges and are still controlled by FASB ASC 830-20-35-1. For these derivatives, the change in fair value is recognized immediately in net income.

On January 1, year 1, a company capitalized $100,000 of costs for software that is to be sold. The company amortizes the software costs on a straight-line basis over five years. The carrying value of the software costs on January 1, year 3, was $60,000. As of December 31, year 3, the estimated future gross revenue to be generated from the sale of the software is $23,000, and the estimated future cost of disposing of the software is $8,000. What amount should the company expense related to the software costs for the year ended December 31, year 3? $18,400 $20,000 $37,000 $45,000

$45,000 Software production costs are capitalized and reported at the lower of unamortized cost or net realizable value (NRV) once technological feasibility has been met. The unamortized cost is $60,000 and the NRV is $15,000 ($23,000 − $8,000); therefore, the software should be written down by $45,000 (i.e., expensed) to the NRV of $15,000.

Baker Co. sells consumer products that are packaged in boxes. Baker offered an unbreakable glass in exchange for two box tops and $1 as a promotion during the current year. The cost of the glass was $2. Baker estimated at the end of the year that it would be probable that 50% of the box tops will be redeemed. Baker sold 100,000 boxes of the product during the current year and 40,000 box tops were redeemed during the year for the glasses. What amount should Baker accrue as an estimated liability at the end of the current year, related to the redemption of box tops? $0 $5,000 $20,000 $25,000

$5,000 A contingent liability must be reported if it is probable that the liability will occur and the amount can be reasonably estimated. Boxes sold 100,000 Estimated redemption percentage x 0.50 Estimated redemptions 50,000 Box tops already redeemed 40,000 Remaining box tops to be redeemed 10,000 Box tops per redemption / 2 Estimated glasses to be provided 5,000 Cost per glass ($2 - $1) $ 1 Estimated liability $ 5,000 This treatment is confirmed in FASB ASC 450-20-25-2(a): "Information available before the financial statements are issued or are available to be issued (as discussed in Section 855-10-25) indicates that it is probable that an asset had been impaired or a liability had been incurred at the date of the financial statements. Date of the financial statements means the end of the most recent accounting period for which financial statements are being presented. It is implicit in this condition that it must be probable that one or more future events will occur confirming the fact of the loss."

Farm Co. leased equipment to Union Co. on July 1, 20X1, and properly recorded the sales-type lease at $135,000, the present value of the lease payments discounted at 10%. The first of eight annual lease payments of $20,000 due at the beginning of each year was received and recorded on July 3, 20X1. Farm had purchased the equipment for $110,000. What amount of interest revenue from the lease should Farm report in its 20X1 income statement? $0 $5,500 $5,750 $6,750

$5,750 Initial amount of lease $135,000 Less first payment 20,000 Lease amount applicable to last half of 20X1 $115,000 Times interest rate (10% x 6/12 year) x .05 Interest revenue for 20X1 $ 5,750 ========

On January 2 of the current year, LTTI Co. entered into a three-year, noncancelable contract to buy up to 1,000,000 units of a product each year at $.10 per unit with a minimum annual guarantee purchase of 200,000 units. At year-end, LTTI had only purchased 80,000 units and decided to cancel sales of the product. What amount should LTTI report as a loss related to the purchase commitment as of December 31 of the current year? $0 $8,000 $12,000 $52,000

$52,000 LTTI had a purchase commitment for 600,000 units (200,000 × 3) and purchased 80,000 units. By canceling sales of the product, LTTI has a loss of $52,000 (520,000 units × .10).

During the year, Pitt Corp. incurred costs to develop and produce a routine, low-risk computer software product, as follows: Design of tools, jigs, molds, and dies involving new technology $125,000 Completion of detail program design 13,000 Costs incurred for coding and testing to establish technological feasibility 10,000 Other coding costs after establishment of technological feasibility 24,000 Other testing costs after establishment of technological feasibility 20,000 Costs of producing product masters for training materials 15,000 Duplication of computer software and training materials from product masters (1,000 units) 25,000 Packaging product (500 units) 9,000 In Pitt's December 31 balance sheet, what amount should be capitalized as software cost, subject to amortization? $54,000 $57,000 $59,000 $69,000

$59,000 The capitalized costs here are generally the costs incurred after attaining technological feasibility. Hence, the coding and testing costs of $24,000 and $20,000 are capitalized, but also the costs of the product masters for training of $15,000. Thus, a total of $59,000 would be capitalized, and the rest would be expensed as research and development.

A corporation is in the final stages of developing a computer software program that will be sold to the general public. The company's costs related to the software are as follows: Development of a working model of the software $4 million Customer support and training 2 million Product master production 1 million The costs associated with the product master production were incurred after the establishment of technological feasibility. What amount, if any, should the corporation expense against earnings? $6 million $5 million $4 million $0

$6 million All of the costs an entity incurs to develop the technological feasibility of computer software are expensed ($4 million + $2 million = $6 million). Technological feasibility has occurred when the software is ready to be sold or leased to customers. Customer support and maintenance costs are also expensed. The costs of producing product masters after technological feasibility is achieved are capitalized.

On December 30 of the current year, Haber Co. leased a new machine from Gregg Corp. The following data relate to the lease transaction at the commencement of the lease: Lease term 10 years Annual rental payable at the end of each lease year $100,000 Estimated life of machine 12 years Implicit interest rate 10% Present value of an annuity of $1 in advance for 10 periods at 10% 6.76 Present value of an annuity of $1 in arrears for 10 periods at 10% 6.15 Fair value of the machine $700,000 The lease has no renewal option, and the possession of the machine reverts to Gregg when the lease terminates. At the commencement of the lease, Haber should record a lease liability of: $0 $615,000 $630,000 $676,000

$615,000 This lease qualifies as a finance lease because the lease term of 10 years exceeds 75% of the asset's useful life of 12 years (10 > 0.75 × 12 = 9). When a lease is considered a finance lease, then the lessee capitalizes the lease property as a right-of-use asset and recognizes a lease obligation for the present value of the lease payments. The lease payments are $100,000 a year at the end of each year, and their present value is $100,000 × 6.15 (present value of an annuity in arrears or ordinary annuity, for 10 periods at 10%). Thus, the answer is $615,000: $100,000 × 6.15 = $615,000

Tableau Company manufactures hot water heater systems that carry a 2-year warranty against defects. Based on past experience, warranty costs are estimated at 2.5% of sales for the warranty period. During 20X9, hot water heater system sales totaled $2,600,000, and warranty costs of $31,300 were incurred. In its income statement for the year ending December 31, 20X9, Tableau should report warranty expense of: $65,000. $60,000. $33,700. $31,300.

$65,000. Warranty expense for 20X9 is $65,000 ($2,600,000 × 0.025). The warranty expense is recognized in the year in which the warranted product is sold. The actual warranty expenditures may or may not be made in that same period. The $31,300 warranty expenditures incurred in 20X9 result in a reduction of the estimated warranty liability. Those expenditures may relate to products sold in 20X9 but they also may relate to products sold in a prior period. Journal entries include the following: Warranty expense 65,000 Estimated warranty liability 65,000 Estimated warranty liability 31,300 Cash 31,300

On December 30, 20X1, Rafferty Corp. leased equipment under a finance lease. Annual lease payments of $20,000 are due December 31 for 10 years. The equipment's useful life is 10 years, and the interest rate implicit in the lease is 10%. The finance lease obligation was recorded on December 30, 20X1, at $135,000, and the first lease payment was made on that date. What amount should Rafferty include in current liabilities for this lease in its December 31, 20X1, balance sheet? $6,500 $8,500 $11,500 $20,000

$8,500 Initial lease obligation on December 31, 20X1 $135,000 Less payment made on December 31, 20X1 - 20,000 Lease obligation during 20X2 $115,000 ======== Portion of December 31, 20X2, payment that is interest = rate x obligation x time = 10% x $115,000 x 1 = $11,500 Portion of December 31, 20X2, payment that is related to lease obligation = payment - interest portion = $20,000 - $11,500 = $8,500 This amount is a current liability since it is payable within the current period. The remaining lease obligation is noncurrent.

On January 1, year 1, a company purchased equipment for $100 million. The equipment consists of four major components, of which two components comprise 80% of the total cost and each has a 20-year useful life. The remaining two components have costs of $10 million each; one of them has a useful life of 4 years, and the other has a useful life of 5 years. The company applies the cost model to the equipment and uses the straight-line method of depreciation. Under IFRS, what is the depreciation expense for the year ended December 31, year 1? $4,000,000 $5,000,000 $8,000,000 $8,500,000

$8,500,000 Under IFRS, component depreciation is required whenever a part of an item is significant in cost compared to the total cost of the item. Component depreciation is computed as follows: ($80,000,000 × 1/20) + ($10,000,000 × 1/4) + ($10,000,000 × 1/5) = $4,000,000 + $2,500,000 + $2,000,000 = $8,500,00

On January 1, 20X3, Warren Co. purchased a $600,000 machine with a 5-year useful life and no salvage value. The machine was depreciated by an accelerated method for book and tax purposes. The machine's carrying amount was $240,000 on December 31, 20X4. On January 1, 20X5, Warren changed to the straight-line method for financial statement purposes. Warren can justify the change. How much depreciation expense for this asset should Warren's report in its 20X5 income statement? $80,000 $100,000 $120,000 $240,000

$80,000 Depreciation expense is computed prospectively, using the revised carrying value and the remaining estimated useful life: $240,000 ÷ 3 years = $80,000 depreciation expense.

West, Inc., acquired 60% of East Co.'s outstanding common stock. West paid $800,000 to acquire the stock. West plans to relocate East's company headquarters, which is expected to cost between $100,000 and $300,000. The present value of the probability-adjusted relocation cost is $240,000. What is West's acquisition cost? $800,000 $900,000 $1,040,000 $1,100,000

$800,000 The acquisition of the stock would be reported at cost—the amount paid for the stock. Future relocation costs would not be included in the current cost of the stock. Costs expected to be incurred are not liabilities and should not be included in the acquisition cost.

Crossroads Co. chooses to report a financial asset at its fair value. The asset trades in two different markets; however, neither market is the principal market for the financial asset. In the first market, sales proceeds are $76, which is net of transaction costs of $6. In the second market, the sales proceeds are $80, which is net of transaction costs of $1. What amount should Crossroads report as the fair value of the asset? $76 $80 $81 $82

$81 Fair value measurement assumes that the transaction to sell the asset or transfer the liability occurs in the principal market for the asset or liability. If there is no principal market, the entity should use the most advantageous market for that asset or liability. The FASB defines the most advantageous market as "the market that maximizes the amount that would be received to sell the asset or minimizes the amount that would be paid to transfer the liability, after taking into account transaction costs and transportation costs" (FASB ASC 820-10-20). The price used to measure fair value should not be adjusted for transaction costs, which are the incremental direct costs to sell the asset or transfer the liability. In other words, in determining the most advantageous market, adjust for transaction costs. But to compute the actual fair value amount, ignore the transaction costs. The first market yields $76 and the second market yields $80, so the second market is the most advantageous. The fair value in the second market is $81 ($80 with the $1 added back).

On January 1, 20X1, JCK Co. signed a contract for an 8-year lease of its equipment with a 10-year life. The present value of the 16 equal semiannual payments in advance equaled 85% of the equipment's fair value. The contract had no provision for JCK, the lessor, to give up legal ownership of the equipment. Should JCK recognize rent or interest revenue in 20X3, and should the revenue recognized in 20X3 be the same or smaller than the revenue recognized in 20X2? 20X3 interest revenue recognized; 20X3 amount recognized smaller than 20X2 20X3 rent revenue recognized; 20X3 amount recognized the same as 20X2 20X3 rent revenue recognized; 20X3 amount recognized smaller than 20X2 20X3 interest revenue recognized; 20X3 amount recognized the same as 20X2

20X3 interest revenue recognized; 20X3 amount recognized smaller than 20X2 JCK is the lessor. The principal issue is whether the lease is an operating lease or qualifies as a "sales-type or direct financing type" lease. To qualify as a "sales-type" lease, it first must meet one of the five criteria specified in FASB ASC 842-10-25-2 applicable to lessees. One of those criteria is that the lease is a finance lease if the lease term is a major part of the estimated economic life of the lease property. Since the 8-year lease term is 80% of the estimated economic life (8 years ÷ 10 years), at least one of those four criteria is met. Thus, JCK should account for the lease as a "sales-type" lease (either direct financing or sales-type). The receipt of the payments will be accounted for under the interest method, which means that the portion of the equal semiannual payments received is recognized as interest revenue each year and the amount recognized in 20X3 will be less than that recognized in 20X2. If the lease had been an operating lease, the payments received would have been recognized as rent revenue (rather than interest revenue) and would have been the same each year.

Douglas Co. leased machinery with an economic useful life of 6 years. For tax purposes, the depreciable life is 7 years. The lease is for 5 years, and Douglas can purchase the machinery at fair market value at the end of the lease. What is the depreciable life of the leased machinery for financial reporting? 0 5 years 6 years 7 years

5 years Douglas Co. should consider the lease as a finance lease because the lease term of 5 years is for the major part of the remaining economic life of 6 years for the underlying asset. In order to use the full economic life of 6 years for depreciation, the FASB requires that it is reasonably certain the option will be exercised; this requires judgment as the "bright line" criteria are no longer required. The purchase option is for fair market value (FMV) for a 5-year-old asset with only 1 year left. (Note that there is no "break" in the price; Douglas can purchase a used asset anywhere at FMV.) In all likelihood, Douglas will not exercise the option but will probably create a new lease for a new asset (with all the technological improvements that have occurred over the last 5 years). Therefore, the best answer choice is using a depreciable life of 5 years.

On December 31, 20X1, Bit Co. had capitalized costs for a new computer software product with an economic life of 5 years. Sales for 20X2 were 30% of expected total sales of the software. On December 31, 20X2, the software had a net realizable value equal to 90% of the capitalized cost. What percentage of the original capitalized cost should be reported as the net amount on Bit's December 31, 20X2, balance sheet? 70% 72% 80% 90%

70% FASB ASC 985-20-35-1 provides: "The annual amortization shall be the greater of the amount computed using (a) The ratio that current gross revenues for a product bear to the total of current and anticipated future gross revenues for that product, or (b) The straight-line method over the remaining estimated economic life of the product including the period being reported on." Ratio of current to total revenues (given) = 30% Straight-line rate = 1/5 = 20% The greater of these, 30%, would be used in computing 20X2 amortization, leaving a net amount of 70% (100% − 30%) to be shown on Bit's December 31, 20X2, balance sheet.

Which of the following accounting changes should be given retroactive treatment when preparing the financial statements? A change in the amount of expected repairs warranty repairs A change in the settlement rate used in computing pension expense A change in the useful life of a finite life intangible asset A change from the weighted-average method to the FIFO method of inventory costing

A change from the weighted-average method to the FIFO method of inventory costing A change in accounting principle is given retroactive treatment when reported in the financial statements. This means that all periods presented need to reflect the new accounting principle. A change in accounting estimate is given prospective treatment, meaning that all changes are reported in current and future periods, but no prior periods are restated. Of the four choices, only the change from weighted average to FIFO for inventory costing qualifies as a change in accounting principle.

On March 1, 20X1, Ila Co. made a significant decision to not exercise the purchase option at the end of the lease term. requiring modification of the terms of a 4-year lease of equipment. Ila had leased the equipment on January 1, 20X1, and properly recorded it as a finance lease due to the option purchase. Under the modified provisions, the lease would have been classified as: a sales-type lease. a finance lease. an operating lease. a leverage lease.

An operating lease FASB ASC 842-10-25-9 requires that a modified lease not accounted for as a separate contract to be reassessed as of the modification date and be accounted for in accordance to FASB ASC 842-10-25-10 through 25-14.

How should the effect of a change in accounting principle that is inseparable from the effect of a change in accounting estimate be reported? As a component of income from continuing operations By restating the financial statements of all prior periods presented As a correction of an error By footnote disclosure only

As a component of income from continuing operations FASB ASC 250-10-45-18 requires that whenever a change in accounting principle is inseparable from a change in an accounting estimate, the change should be considered as a change in estimate. Changes in estimates are handled prospectively. That is, previously reported information in previous financial statements is not adjusted, nor is a cumulative effect of the change reported. Prospective treatment only requires utilization of the change(s) in the current period as it effects the current period's income. It is part of income from continuing operations because no special disclosure is required on the face of the income statement under the prospective approach.

How should the acquirer recognize a bargain purchase in a business acquisition? As negative goodwill in the statement of financial position As goodwill in the statement of financial position As a gain in earnings at the acquisition date As a deferred gain that is amortized into earnings over the estimated future periods benefited

As a gain in earnings at the acquisition date In a business acquisition, the acquiring corporation must recognize the assets and liabilities acquired at fair value. If the net assets acquired exceed the purchase price—a bargain purchase—the excess must be recognized as a gain in earnings at the date of the acquisition.

In a business combination with goodwill recorded, how should any subsequent impairment of the goodwill be recognized on the income statement or statement of retained earnings? As a component of other comprehensive income As a change in accounting principle As a loss from continuing operations As a restatement of beginning retained earnings

As a loss from continuing operations The impairment loss should be recognized in income from continuing operations just as amortization expense would have been recognized in arriving at income from continuing operations. (FASB ASC 350-20-45-1)

Lease M does not contain a purchase option, but the lease term is equal to 90% of the estimated economic life of the leased property. Lease P does not transfer ownership of the property to the lessee at the end of the lease term, but the lease term is equal to 75% of the estimated economic life of the leased property. How should the lessee classify these leases? Lease M as a finance lease and Lease P as an operating lease Lease M as an operating lease and Lease P as a finance lease Both Lease M and Lease P as finance leases Both Lease M and Lease P as operating leases

Both Lease M and Lease P as finance leases FASB ASC 842-10-25-2 established five criteria for classifying leases. The first set of criteria result in a finance lease for a lessee or sales-type lease for the lessor if the lease meets any of the following criteria at commencement: Title (ownership) transfers to the lessee by the end of the lease term. Lease contains a purchase option that the lessee is reasonably certain to exercise. The lease term is for the major part of the remaining economic life of the underlying asset. This criterion shall not be used if the lease commencement date is near the end of the asset's economic life. The present value of the sum of the lease payments and any lessee guaranteed residual value not already in the lease payments equals or exceeds substantially all of the fair value of the underlying asset. The underlying asset is specialized and is not expected to have an alternative use to the lessor at the end of the lease term. If one or more of these conditions are present, the lease is a finance lease. The FASB has removed the former bright-line tests but in FASB ASC 842-10-55-2 uses the former rules as benchmarks. Lease M is a finance lease because the lease term is greater than 75% of the economic life of the property. Lease P is also a finance lease for the same reason.

An entity, upon initial recognition of an asset retirement obligation, should not take which of the following actions? Allocate asset retirement cost to expense over the useful life of the related asset. Measure the asset retirement cost at fair value. Capitalize the asset retirement cost by increasing the carrying amount of the related asset. Capitalize the asset retirement cost at its undiscounted cash flow value.

Capitalize the asset retirement cost at its undiscounted cash flow value. An asset retirement obligation (ARO) refers to an obligation associated with the retirement of a tangible, long-lived asset, such as a nuclear power plant. An entity should recognize the fair value of an ARO in the period in which it is incurred if a reasonable estimate of fair value can be made. Otherwise, the liability should be recognized in the first period in which a reasonable estimate of fair value can be made. When an ARO is initially recognized, an entity should capitalize the ARO cost by increasing the carrying amount of the related long-lived asset by the same amount as the ARO. Subsequently, the entity should amortize the ARO to expense using a systematic and rational method over its useful life. An entity should not capitalize the ARO at its undiscounted cash flow value.

Which of the following methods should be used to account for research and development costs with no alternative future use? Charging all costs to expense when incurred Capitalizing all costs when incurred Capitalizing costs specified by management and charging all other costs to expense Accumulating all costs in a separate component of stockholders' equity until the existence of future benefits can be determined

Charging all costs to expense when incurred The future benefits associated with research and development (R&D) expenses are expenditures that are uncertain; therefore, GAAP requires that all R&D be charged to expense as incurred. There are limited exceptions to the rule, including capitalization of the expenses when the cost relates to tangible assets which have alternative future uses.

At the end of year 1, a company reduced its inventory cost from $100 to its net realizable value of $80. As of the end of year 2, the inventory was still on hand and its net realizable value increased to $150. Under IFRS, what journal entry should the company record for year 2 to properly report the inventory value? Debit inventory for $20 and credit expense for $20 Debit inventory for $70 and credit expense for $70 Debit inventory for $70, credit retained earnings for $50, and credit expense for $20 Debit inventory for $20, debit expense for $30, and credit retained earnings for $50

Debit inventory for $20 and credit expense for $20 Under IFRS, inventory is carried at the lower of cost or net realizable value (NRV). This situation typically arises when inventory has deteriorated, has become obsolete, or market prices have declined. Write-downs taken to reduce inventories to the lower of cost or NRV are reversed for subsequent increases in value. At the end of year 2, cost was $100 and NRV was $150; inventory should be carried at the lower amount of $100. Since the inventory was previously written down to $80, the year 2 journal entry is: Inventory 20 Inventory Expense 20 Some companies prefer to use an allowance account for inventory write-downs and recoveries. Under those circumstances, the year 2 journal entry would be: Allowance to Reduce Inventory to NRV 20 Recovery of Inventory Loss 20 IAS 2.33 requires that any write-down of inventory to NRV is recognized as an expense in the period in which it occurs. Any subsequent increases in NRV of inventory previously written down are recognized as a reduction on inventory expense in the period in which they occur (up to the amount of the original write-down).

Potter Co. has the following contingencies, all resulting from lawsuits in progress during the current year: Probable loss contingency $1,500,000 Reasonably possible loss contingency 500,000 Probable gain contingency 700,000 Reasonably possible gain contingency 300,000 Potter's accountant believes the financial statements will be misleading if the probable loss contingency is not disclosed. How much should be disclosed, and how much should be accrued in Potter's financial statements for the current year? Disclosed: $500,000 loss and $1,000,000 gain; Accrued: $1,500,000 loss and $700,000 gain Disclosed: $2,000,000 loss and $1,000,000 gain; Accrued: $1,500,000 loss Disclosed: $1,000,000 gain; Accrued: $1,500,000 loss and $500,000 loss Disclosed: $500,000 loss and $300,000 gain; Accrued: $1,500,000 loss

Disclosed: $2,000,000 loss and $1,000,000 gain; Accrued: $1,500,000 loss Contingent losses are accrued when probable and reasonably estimable. Contingent losses are disclosed when probable or reasonably probable. Contingent gains are not accrued but should be disclosed when probable or reasonably probable. Only the reasonably probable loss should be accrued ($1,500,000). All of the gains and losses should be disclosed (Losses: $1,500,000 + $500,000 = $2,000,000, Gains: $700,000 + $300,000 = $1,000,000).

Conlon Co. is the plaintiff in a patent infringement case. Conlon has a high probability of a favorable outcome, and can reasonably estimate the amount of the settlement. What is the proper accounting treatment of the patent infringement case? A gain contingency for the minimum estimated amount of the settlement A gain contingency for the estimated probable settlement Disclosure in the notes only No reporting is required at this time.

Disclosure in the notes only Since Conlon is the plaintiff (they are suing), a gain contingency exists. Gain contingencies are not recognized since they have not been realized (conservatism).

Town, Inc., is preparing its financial statements for the year ended December 31, 20X1. On January 5, 20X2, prior to the issuance of the financial statements, Town redeemed its outstanding bonds and issued new bonds with a lower rate of interest. The reacquisition price was in excess of the carrying amount of the bonds. What is the appropriate reporting requirement? Disclosure only Accrual only Both accrual and disclosure Neither accrual nor disclosure

Disclosure only Information which becomes known after the balance sheet date, but before the financial statements are issued, should be disclosed to keep the financial statements from being misleading. The gains or losses associated with the redemption and issuance of the bonds would be reported and disclosed in the 20X2 financial statements.

Tally Inc. recently signed a lease for office computer equipment. Since each computer is under the $5,000 recognition exemption, Tally does not have to capitalize the leased assets. Tally must prepare its financial statements in accordance with: IFRS. U.S. GAAP. either IFRS or U.S. GAAP. neither IFRS nor U.S. GAAP.

IFRS IFRS 16, Leases, includes a specific recognition exemption for low-value lease assets (approximately $5,000) such as cell phones and computers; these leases can be expensed rather than capitalized. FASB ASC Topic 842, Leases, does not list a specific cost threshold, although U.S. GAAP does permit "immaterial" assets to be expensed rather than capitalized.

A company's research department incurred $1,000,000 in material, labor, and overhead costs to construct a prototype of a new product and $100,000 to test and modify the prototype. Which of the following statements correctly describes the accounting treatment of prototype costs incurred by the company? Capitalize $1,100,000 and amortize it over the expected sales life of the new product. Capitalize $1,100,000 and amortize it over the life of the prototype. Capitalize $1,000,000 and amortize it over the life of the prototype and expense $100,000 as incurred. Expense $1,100,000 as incurred.

Expense $1,100,000 as incurred. All research and development costs are expensed when incurred. A useful way to determine if costs are research and development costs is to establish if they relate to activities identified with the period prior to the beginning of commercial production. Research and development costs are identified in five categories: (1) materials, equipment, and facilities; (2) personnel; (3) intangibles; (4) contract services; and (5) indirect costs. A prototype is a preliminary or first model, often built for demonstration purposes (not production) from which other forms are copied or developed. Both the $1,000,000 and the $100,000 qualify as R&R cost and should therefore be expensed as incurred. The remaining answer choices are all incorrect as the items cannot be capitalized.

Which of the following expenses related to the business combination should be included, in total, in the determination of net income of the combined corporation for the period in which the expenses are incurred? Fees of finders and consultants: Yes; Issuance fees for equity securities issues: Yes Fees of finders and consultants: Yes; Issuance fees for equity securities issues: No Fees of finders and consultants: No; Issuance fees for equity securities issues: Yes Fees of finders and consultants: No; Issuance fees for equity securities issues: No

Fees of finders and consultants: Yes; Issuance fees for equity securities issues: No Business combinations accounted for as an acquisition should treat expenses related to the combination as follows: Out-of-pocket costs such as fees of finders and consultants are expensed. Issuance costs such as SEC filing fees are charged to the paid-in-capital account. FASB ASC 805-20-25-2 states the following: "Recognition Conditions: To qualify for recognition as part of applying the acquisition method, the identifiable assets acquired and liabilities assumed must meet the definitions of assets and liabilities in FASB Concepts Statement No. 6, Elements of Financial Statements, at the acquisition date. For example, costs the acquirer expects but is not obligated to incur in the future to effect its plan to exit an activity of an acquiree or to terminate the employment of or relocate an acquiree's employees are not liabilities at the acquisition date. Therefore, the acquirer does not recognize those costs as part of applying the acquisition method. Instead, the acquirer recognizes those costs in its postcombination financial statements in accordance with other applicable generally accepted accounting principles (GAAP)." FASB ASC 805-10-25-23 states the following: "Acquisition-Related Costs: Acquisition-related costs are costs the acquirer incurs to effect a business combination. Those costs include finder's fees; advisory, legal, accounting, valuation, and other professional or consulting fees; general administrative costs, including the costs of maintaining an internal acquisitions department; and costs of registering and issuing debt and equity securities. The acquirer shall account for acquisition-related costs as expenses in the periods in which the costs are incurred and the services are received, with one exception. The costs to issue debt or equity securities shall be recognized in accordance with other applicable GAAP." (Emphasis added)

Neron Co. has two derivatives related to two different financial instruments, Instrument A and Instrument B, both of which are debt instruments. The derivative related to Instrument A is a fair value hedge, and the derivative related to Instrument B is a cash flow hedge. Neron experienced gains in the value of Instruments A and B due to a change in interest rates. Which of the gains should be reported by Neron in its income statement? Gain in value of both debt Instruments A and B Gain in value of debt Instrument A only Gain in value of debt Instrument B only Neither gain in value of debt Instrument A or B

Gain in value of debt Instrument A only FASB ASC 815-25-35-1 requires that gains or losses associated with changes in the fair value of the hedging instrument be recognized in net income in the period in which the change in fair value takes place. The gain or loss resulting from changes in the fair value of a cash flow hedge is included in other comprehensive income. Consequently, only the gain in the value of Instrument A would be included in net income.

Which of the following statements related to initial direct costs (IDC) is correct? IDC include items such as commissions and payments made to current tenants to obtain the lease. IDC include all internal costs associated with obtaining the lease. IDC are included by the lessor in the measurement of an ROU asset. IDC are expensed as incurred.

IDC include items such as commissions and payments made to current tenants to obtain the lease. IDC are capitalized (not expensed) and include only those costs that entity would not have incurred if the lease had not been entered into (i.e., must be incremental costs, not internal, such as commissions and payments made to current tenants to obtain the lease). The lessee (not the lessor) includes IDC in the measurement of a right-of-use (ROU) asset.

Packet Corp. is in the process of preparing its financial statements for the year ended December 31, 20X1. How would a gain on remeasuring a foreign subsidiary's financial statements from the local currency into the functional currency that occurred during 20X1 be classified in these financial statements? Income from continuing operations, with no separate disclosure Income from continuing operations, with separate disclosure (either on the face of statement or in the notes) Other comprehensive income None of the answer choices are correct.

Income from continuing operations, with separate disclosure (either on the face of statement or in the notes) FASB ASC 830-30-45-17 provides: "It is also necessary to recognize currently in income all exchange gains and losses from remeasurement of monetary assets and liabilities." FASB ASC 830-30-45-18 states: "An analysis of the changes...shall be provided in a separate financial statement, in notes to the financial statements, or as a part of a statement of changes in equity."

Under IFRS, which of the following statements about intangible assets is correct? Internally generated goodwill cannot be recognized as an asset. Intangible assets within a class may be measured differently using either the cost model or the revaluation model. Research and development costs are capitalized as incurred. Intangible assets with indefinite lives must be amortized annually.

Internally generated goodwill cannot be recognized as an asset. IFRS (International Financial Reporting Standards) explicitly states that internally generated goodwill shall not be recognized as an asset. Intangible assets within a class must all be measured using the same model; different classes can use different models. Research and development costs are expensed until economic viability is reached, then they are capitalized, similar to R&D treatment under GAAP. An intangible asset that has an indefinite life should be reviewed annually for impairment, but it is not amortized until its useful life is determined to no longer be indefinite; from that point on, it should be amortized on a prospective basis.

On January 1, Year 2, a finance lease agreement was entered into that requires annual lease payments of $42,500 over a 4-year lease term, which equals the remaining useful life of the asset. The first payment is due on January 1 of Year 2 and every December 31 thereafter. The interest rate is 7%. What would be the amount of the lease liability and interest expense, respectively, reported on the December 31 balance sheet and income statement, respectively, at the end of Year 2? PV Factors: Annuity Due n=3; i=7 2.80800 Annuity Due n=4; i=7 3.62432 Ordinary Annuity n=3; i=7 2.62432 Ordinary Annuity n=4; i=7 3.38721 Lease Liability = $111,533; Interest Expense = $0 Lease Liability = $76,840; Interest Expense = $7,807 Lease Liability = $122,316; Interest Expense = $10,782 Lease Liability = $66,058; Interest Expense = $7,102

Lease Liability = $76,840; Interest Expense = $7,807 The original liability can be calculated in one of two ways: (1) present value of an annuity due for 4 periods at 7% or (2) present value of an ordinary annuity for 3 periods plus the first payment. The answer is exactly the same. Present Value, Annuity Due (PVAD): $42,500 at 7% for 4 periods = $42,500 × 3.62432 = $154,033 Present Value, Ordinary Annuity (PVOA): $42,500 at 7% for 4 periods = $42,500 × 2.62432 = $111,533 + $42,500 = $154,033 One payment is made on day one, reducing the lease liability before any interest ($154,033 - $42,500 = $111,533). Interest on the liability for the year would be $7,807 ($111,533 × .07). Another payment was made on the last day of the year, so the liability would be $76,840 with rounding ($111,533 + $7,807 - $42,500).

A company leases trucks and properly classifies the leases as finance leases. The leases have a 10-year term, and the lease calculations were done 3 years ago when interest rates were lower. Which of the following is the appropriate accounting treatment, if any, for the application of the fair value option to lease transactions? Leases are not eligible for the fair value option. Recognize the change to fair value accounting with a cumulative adjustment to beginning retained earnings. Recognize the change to fair value accounting with an unrealized loss in the income statement. Recognize the change to fair value accounting with an unrealized loss in accumulated other comprehensive income.

Leases are not eligible for the fair value option. Companies may choose to measure a wide range of financial assets and liabilities at fair value. Even though leases are considered financial in nature, the FASB specifically excludes "financial assets and financial liabilities recognized under leases" from the fair value option of accounting (FASB ASC 825-10-15-5). The remaining answer choices are not correct as there is no option to recognize the change in fair value.

Which of the following statements concerning patents is correct? Research and development costs incurred to develop a patented item should be capitalized and amortized on a straight-line basis over 17 years. Legal costs incurred to successfully defend an internally developed patent should be capitalized and amortized over the patent's remaining economic life. Research and development contract services purchased from others and used to develop a patented manufacturing process should be capitalized and amortized over the patent's economic life and not assessed for impairment. Legal fees and other direct costs incurred in registering a patent should be capitalized and amortized on a straight-line basis over a 5-year period.

Legal costs incurred to successfully defend an internally developed patent should be capitalized and amortized over the patent's remaining economic life. In general, the cost of internally developed patents should be expensed in the period incurred. An exception to this is the treatment of legal costs related to patents as required by FASB ASC 730-10-55-2. In a listing of examples of activities to be excluded from research and development treatment (i.e., expensed when incurred) is "i. Legal work in connection with patent applications or litigation, and the sale or licensing of patents." This means that legal costs related to the successful defense of internally developed patents should be capitalized and amortized over the patent's remaining economic life. FASB ASC 350-30-35-14 requires that intangibles subject to amortization also be assessed for impairment.

Which of the following subsequent events must not be recognized in the financial statements? Loss on an uncollectible trade account receivable as a result of a customer's deteriorating financial condition leading to bankruptcy after the balance sheet date but before the financial statements are issued or are available to be issued Loss of plant or inventories as a result of fire or natural disaster that occurred after the balance sheet date but before financial statements are issued or are available to be issued The events that gave rise to litigation took place before the balance sheet date and that litigation is settled, after the balance sheet date but before the financial statements are issued or are available to be issued, for an amount different from the liability recorded in the accounts. Shortly before financial statements are issued, the actual loss of plant or inventories as a result of fire or natural disaster that occurred before the balance sheet date is determined to be greater than the loss that was originally estimated.

Loss of plant or inventories as a result of fire or natural disaster that occurred after the balance sheet date but before financial statements are issued or are available to be issued The correct answer is "loss of plant or inventories as a result of fire or natural disaster that occurred after the balance sheet date but before financial statements are issued or are available to be issued." An entity must not recognize subsequent events that provide evidence about conditions that did not exist at the date of the balance sheet but arose after the balance sheet date but before financial statements are issued or are available to be issued. The other answer choices are incorrect because an entity must recognize in the financial statements the effects of all subsequent events that provide additional evidence about conditions that existed at the date of the balance sheet.

Which of the following subsequent events must be recognized in the financial statements? Loss on an uncollectible trade account receivable as a result of a customer's deteriorating financial condition leading to bankruptcy after the balance sheet date but before the financial statements are issued or are available to be issued Sale of a bond or capital stock issued after the balance sheet date but before financial statements are issued or are available to be issued Changes in the fair value of assets or liabilities (financial or nonfinancial) or foreign exchange rates after the balance sheet date but before financial statements are issued or are available to be issued Settlement of litigation when the event giving rise to the claim took place after the balance sheet date but before financial statements are issued or are available to be issued

Loss on an uncollectible trade account receivable as a result of a customer's deteriorating financial condition leading to bankruptcy after the balance sheet date but before the financial statements are issued or are available to be issued The correct answer is "loss on an uncollectible trade account receivable as a result of a customer's deteriorating financial condition leading to bankruptcy after the balance sheet date but before the financial statements are issued or are available to be issued." An entity must recognize in the financial statements the effects of all subsequent events that provide additional evidence about conditions that existed at the date of the balance sheet. The other answer choices are incorrect because an entity must not recognize subsequent events that provide evidence about conditions that did not exist at the date of the balance sheet but arose after the balance sheet date but before financial statements are issued or are available to be issued.

Which is the correct way to compute periodic interest expense by a lessee for a finance lease? Multiply the carrying amount of the lease liability by the lease's discount rate Multiply the straight-line lease expense by the lease's discount rate Multiply the periodic ROU asset amortization expense by the lease's discount rate Multiply the undiscounted cash flows over the term of the lease by the lease's discount rate

Multiply the carrying amount of the lease liability by the lease's discount rate Finance leases use the effective interest method, which calculates interest expense by multiplying the discount rate times the carrying value.

On November 1 of the current year, Fruita Corporation entered into an option contract to purchase 800 shares of Silt Company stock for $19 per share (the same as the current market price) by the end of the next three months. The time value of the option contract is $500. At the end of December, Silt's stock was selling for $23, and the time value of the option is now $120. If Fruita does not exercise its option until January of the subsequent year, which of the following changes would reflect the proper accounting treatment for this transaction on Fruita's December 31, year-end financial statements? Current assets will decrease by $380. Net income will increase by $2,820. The option value will be disclosed in the footnotes only. Other comprehensive income will increase by $3,200.

Net income will increase by $2,820. The options in this situation are being used as a tool to earn a return on investment, not for hedging purposes. For that reason, the options do not qualify for hedge accounting. The gain or loss must be currently recognized in net income. $23 x 800 = $18,400; $18,400 + $120 = $18,520 $19 x 800 = $15,200; $15,200 + $500 = 15,700 Gain = $ 2,820

On December 1 of the current year, Bann Co. entered into an option contract to purchase 2,000 shares of Norta Co. stock for $40 per share (the same as the current market price) by the end of the next two months. The time value of the option contract is $600. At the end of December, Norta's stock was selling for $43, and the time value of the option is now $400. If Bann does not exercise its option until January of the subsequent year, which of the following changes would reflect the proper accounting treatment for this transaction on Bann's December 31, year-end financial statements? The option value will be disclosed in the footnotes only. Other comprehensive income will increase by $6,000. Net income will increase by $5,800. Current assets will decrease by $200.

Net income will increase by $5,800. Options do not qualify for hedge accounting. The gain or loss must be currently recognized. $43 x 2,000 = $86,000; $86,000 + $400 = $86,400 $40 x 2,000 = $80,000; $80,000 + $600 = 80,600 Gain = $ 5,800

Which of the following is the characteristic of a perfect hedge? No possibility of future gain or loss No possibility of future gain only No possibility of future loss only The possibility of future gain and no future loss

No possibility of future gain or loss The purpose of a hedge is to reduce exposure to a particular type of risk. A perfect hedge would remove all of the risk—that is, remove the possibility of any future gain or loss.

A combination is accounted for as an acquisition. Which of the following would be considered part of the acquisition cost of an acquired entity in a business combination? Costs incurred by the acquiring entity that are directly related to the acquisition Costs incurred by the acquired entity that are directly related to the acquisition Indirect acquisition costs incurred by the acquiring entity I only I and II only I and III only None of these items would be part of the acquisition cost.

None of these items would be part of the acquisition cost. FASB ASC 805-10-25-21 requires that acquisition-related costs be charged to expense. All of these costs are acquisition-related costs and should be expensed in the period incurred. FASB ASC 805-10-25-23 states the following: "Acquisition-related costs are costs the acquirer incurs to effect a business combination. Those costs include finder's fees; advisory, legal, accounting, valuation, and other professional or consulting fees; general administrative costs, including the costs of maintaining an internal acquisitions department; and costs of registering and issuing debt and equity securities. The acquirer shall account for acquisition-related costs as expenses in the periods in which the costs are incurred and the services are received, with one exception. The costs to issue debt or equity securities shall be recognized in accordance with other applicable GAAP."

Wildcat Oil intends to sell 20,000 barrels of crude oil on July 1, 20X6. To hedge against a possible decline in the selling price of crude oil, Wildcat buys a 6-month futures contract on January 1, 20X6, to sell 20,000 barrels of crude oil for $60 per barrel. On June 30, 20X6, the price of crude oil has dropped to $54 per barrel. What adjusting entry does Wildcat record on June 30, 20X6? Futures Contract Expense 120,000 Unrealized holding gain-equity 120,000 Unrealized holding loss 120,000 Futures Contract 120,000 Option (Futures Contract) 120,000 Unrealized holding gain-equity 120,000 No entry is made.

Option (Futures Contract) 120,000 Unrealized holding gain-equity 120,000 Since the price of crude oil has fallen, the value of the futures contract (the derivative) increased in value. Since the contract was a 6-month contract, it expires on June 30, 20X6, and has a fair value just before it expires of $120,000 ($1,200,000 price of crude oil on January 1, 20X6, less $1,080,000 contract price of the futures contract). That $120,000 increase in fair value of the derivative results in a debit to the Futures Contract account (an asset) of $120,000 and a credit to unrealized gain of $120,000. Since the futures contract serves as a hedge of an "anticipated" transaction (the probable sale of 20,000 barrels of crude oil on July 1, 20X6), if it satisfies all other conditions for a hedge, the derivative represents a cash flow hedge rather than a fair value hedge. Accordingly, the unrealized gain of $120,000 would be included in other comprehensive income rather than in net income.

Spring Corp. entered into a 5-year lease agreement with Fall Corp. Spring, the lessee, paid an additional $5,000 nonrefundable lease bonus to Fall upon signing the operating lease agreement. When would Spring recognize in expense the nonrefundable lease bonus paid by Fall? When received Over the life of the lease At the expiration of the lease At the commencement of the lease

Over the life of the lease Fees associated with acquiring a contract—including a lease contract—are considered initial direct costs under and are capitalized in the right-of-use asset and amortized over the life of the lease.

Both quantitative and qualitative disclosures are required for both the lessor and lessee. Which of the following is an incorrect matching of the information and the type of disclosure? Quantitative: information about assets under operating leases Quantitative: information about risks associated with residual values Qualitative: a reconciliation of opening and closing right-of-use (ROU) asset balances Qualitative: a general description of the leasing arrangement

Qualitative: a reconciliation of opening and closing right-of-use (ROU) asset balances Lease disclosure requirements are quite extensive for both the lessor and lessee. The guiding objective is that lessees and lessors provide disclosures that enable users of financial statements to assess the amount, timing, and uncertainty of cash flows arising from leases. Information disclosed is both qualitative and quantitative. Qualitative information describes the quality of something, while quantitative describes the quantity (i.e., amount) of something. A reconciliation is quantitative in nature, not qualitative.

Which of the following statements is correct regarding remeasurement and translation gains and losses? Both remeasurement and translation gains and losses are reported in the income statement in the period they occur. Remeasurement gains and losses are reported in the income statement in the period they occur and translation gains and losses are reported in other comprehensive income. Remeasurement gain and losses are reported in other comprehensive income and translation gains and losses are reported in the income statement in the period they occur. Both remeasurement and translation gains and losses are reported in other comprehensive income.

Remeasurement gains and losses are reported in the income statement in the period they occur and translation gains and losses are reported in other comprehensive income. FASB ASC 830-10-45-17 provides that remeasurement for transactions denominated in a currency other than the functional currency will give rise to a "foreign currency transaction gain or loss that generally shall be included in determining net income for the period in which the exchange rate changes." FASB ASC 830-30-45-12 states, "Translation adjustments shall not be included in determining net income but shall be reported separately and accumulated in comprehensive income."

Which of the following represents a difference in lessor accounting for sales-type versus direct financing leases? Sales-type leases derecognize the leased asset; direct financing leases do not derecognize the leased asset. Selling profit or loss is recognized immediately in a sales-type lease while it is deferred in a direct financing lease. Direct financing leases derecognize the leased asset; sales-type leases do not derecognize the leased asset. In both types, profit or loss on sale is deferred. In a sales-type lease it is recognized over the term of the lease; in a direct financing lease it is recognized at the end of the lease.

Selling profit or loss is recognized immediately in a sales-type lease while it is deferred in a direct financing lease. Both sales-type leases and direct financing leases require the lessor at commencement to derecognize the leased asset and recognize the net investment in the lease. Sales-type leases recognize a selling profit at commencement whereas selling profits are deferred in direct financing leases.

Which of the following items is a required disclosure regarding fair value hedges? The net amount of gains or losses included in the cumulative translation adjustment during the reporting period The amount of net gain or loss recognized in earnings when a hedged firm commitment no longer qualifies as a fair value hedge A description of the transactions or other events that will result in the reclassification into earnings of gains and losses that are reported in accumulated other comprehensive income The estimated net amount of the existing gains or losses at the reporting date that is expected to be reclassified into earnings within the next 12 months

The amount of net gain or loss recognized in earnings when a hedged firm commitment no longer qualifies as a fair value hedge Of the answer choices listed, only "the amount of net gain or loss recognized in earnings when a hedged firm commitment no longer qualifies as a fair value hedge" is a disclosure requirement for a fair value hedge. The other answer choices are disclosure requirements for a cash flow hedge. FASB ASC 815-10-50-4C

Which of the following statements about nonlease components is incorrect? If the component cost represents a transfer of a good or service to the lessee, it is treated as a nonlease component. The lessee includes the nonlease component when computing the present value of the lease payments. Nonlease components are expensed by the lessee. The lessee allocates the consideration to lease and nonlease components on a relative standalone price basis.

The lessee includes the nonlease component when computing the present value of the lease payments. The lessee excludes (not includes) the nonlease component when computing the present value of the lease payments. The other answer choices are correct: If the component cost represents a transfer of a good or service to the lessee, it is treated as a nonlease component. Nonlease components are expensed by the lessee. The lessee allocates the consideration to lease and nonlease components on a relative standalone price basis.

Which of the following statements is correct for the accounting of initial direct costs? The lessor defers the cost and expenses over the life of the lease for both an operating lease and a sales-type lease with no selling profit. The lessee records as an expense at the beginning of the lease. The lessee defers the cost and expenses over the life of the lease for both an operating lease and a sales-type lease with selling profit. The lessor defers the cost and expenses over the life of the lease, but only for a sales-type lease with selling profit.

The lessor defers the cost and expenses over the life of the lease for both an operating lease and a sales-type lease with no selling profit. Lessees defer and then amortize initial direct costs for both operating and finance leases. Lessees do not use a sales-type classification, only operating and finance classifications. Lessors recognize initial direct costs at inception for sales-type leases with profit but defer initial direct costs for operating, direct financing, and sales-type without profit leases.

Which of the following statements is correct related to finance leases? Most leases consist of property. Interest expense related to the discount and amortization related to the ROU asset are combined into a single lease expense. The ROU asset and the lease liability are initially measured at fair value. The lessor is required to derecognize the underlying asset.

The lessor is required to derecognize the underlying asset. The statements "Most leases consist of property" and "Interest expense related to the discount and amortization related to the right-of use (ROU) asset are combined into a single lease expense" are true for operating leases, not finance leases. The right-of-use (ROU) asset and lease liability are initially measured at the present value of the lease payments, not fair value. Only the statement "The lessor is required to derecognize the underlying asset" is correct.

For sales-type leases, which of the following statements is incorrect? The lessor recognizes interest revenue at the beginning of the lease term. The lessor recognizes selling profit at the beginning of the lease term. The fair value of the asset must exceed the cost or carrying value of the asset for selling profit recognition. Selling profit is the difference between sales revenue and cost of goods sold (essentially the same as gross profit).

The lessor recognizes interest revenue at the beginning of the lease term. The lessor recognizes interest revenue over the lease term, not at the beginning of the lease term. The other answer choices are correct: The lessor recognizes selling profit at the beginning of the lease term. The fair value of the asset must exceed the cost or carrying value of the asset for selling profit recognition. Selling profit is the difference between sales revenue and cost of goods sold (essentially the same as gross profit).

For operating leases, which of the following statements is incorrect? An operating lease is considered a "non-debt" liability. The lessor records a lease receivable. An operating lease does not meet any of the criteria for a finance lease. Rights and responsibilities of ownership are retained by the lessor.

The lessor records a lease receivable. The lessor does not record a receivable; rather, the lessor treats operating leases as a rental arrangement and records rent revenue on a straight-line basis over the lease term. Operating leases are considered "non-debt" liabilities to separate them from traditional liabilities, and are therefore excluded from debt ratios. Operating leases cannot meet any of the finance lease criteria, and the rights and responsibilities do not transfer to the lessee but are retained by the lessor.

Which of the following is not one of the criteria for classification as a finance lease? Transfer of ownership Purchase option reasonably certain to exercise No alternative use for the asset The present value of the lease payments being a major part of the fair value

The present value of the lease payments being a major part of the fair value The present value of the lease payments being substantially all (not a major part) of the fair value is one of the five finance lease criteria. The five FASB criteria for a finance lease are (1) transfer of ownership, (2) purchase option reasonably certain to exercise, (3) the lease term is the major part of the economic life of the asset, (4) the present value of the lease payments is substantially all of fair value, and (5) there is no alternative use for the asset.

Which of the following is an eligible item for the fair value measurement option under FASB ASC 825-10-15-4? An investment in a subsidiary that the entity is required to consolidate The rights and obligations under an insurance contract that is not a financial instrument (because it requires or permits the insurer to provide goods or services rather than a cash settlement) but whose terms permit the insurer to settle by paying a third party to provide those goods or services Employers' and plans' obligations (or assets representing net overfunded positions) for pension benefits or other postretirement benefits None of the answer choices are eligible items.

The rights and obligations under an insurance contract that is not a financial instrument (because it requires or permits the insurer to provide goods or services rather than a cash settlement) but whose terms permit the insurer to settle by paying a third party to provide those goods or services FASB ASC 825-10-15-4 lists the following items that are eligible for the fair value election: "A recognized financial asset and financial liability, except any listed in the following paragraph "A firm commitment that would otherwise not be recognized at inception and that involves only financial instruments (An example is a forward purchase contract for a loan that is not readily convertible to cash. That commitment involves only financial instruments—a loan and cash—and would not otherwise be recognized because it is not a derivative instrument.) "A written loan commitment "The rights and obligations under an insurance contract that is not a financial instrument (because it requires or permits the insurer to provide goods or services rather than a cash settlement) but whose terms permit the insurer to settle by paying a third party to provide those goods or services "The rights and obligations under a warranty that is not a financial instrument (because it requires or permits the warrantor to provide goods or services rather than a cash settlement) but whose terms permit the warrantor to settle by paying a third party to provide those goods or services "A host financial instrument resulting from the separation of an embedded nonfinancial derivative instrument from a nonfinancial hybrid instrument under [FASB ASC] 815-15-25-1, subject to the scope exceptions in paragraph 8. (An example of such a nonfinancial hybrid instrument is an instrument in which the value of the bifurcated embedded derivative is payable in cash, services, or merchandise but the debt host is payable only in cash.)"

On January 1, Year 1, a shipping company sells a boat and leases it from the buyer in a sale/leaseback transaction. At the end of the 10-year lease, ownership of the boat reverts to the shipping company. The fair value of the boat, at the time of the transaction, was less than its undepreciated cost. Which of the following outcomes most likely will result from the sale/leaseback transaction? The boat will not be classified in property, plant, and equipment of the shipping company. The shipping company will recognize the total profit on the sale of the boat in the current year. The shipping company will not recognize depreciation expense for the boat in the current year. The shipping company will not derecognize the asset in the current year.

The shipping company will not derecognize the asset in the current year. The first step is to determine if this transaction qualifies as a sale. Per FASB ASC 842-40-25-2, a sale has not occurred if the leaseback could be classified as a finance lease or a sales-type lease. Since title will transfer back to the shipping company, this does not qualify for sale/leaseback treatment. The shipping company will not derecognize the asset and will recognize the consideration received as a financial liability.

Which of the following phrases best describes a Level 1 input for measuring the fair value of an asset or liability? Inputs for the asset or liability based on the reporting entity's internal data Quoted prices for similar assets or liabilities in active markets Inputs that are principally derived from or corroborated by observable market data Unadjusted quoted prices for identical assets or liabilities in active markets

Unadjusted quoted prices for identical assets or liabilities in active markets The FASB's fair value hierarchy prioritizes the inputs to valuation techniques used to measure fair value into three broad levels, as follows: Level 1 inputs—quoted prices (unadjusted) in active markets for identical assets or liabilities that the reporting entity has the ability to access at the measurement date Level 2 inputs—inputs other than quoted prices included within Level 1 that are observable for similar assets or liabilities, either directly or indirectly Level 3 inputs—unobservable inputs for the asset or liability FASB ASC 820-10-35-37

Miller Co. discovered that in the prior year, it failed to report $40,000 of depreciation related to a newly constructed building. The depreciation was computed correctly for tax purposes. The tax rate for the current year was 40%. What was the impact of the error on Miller's financial statements for the prior year? Understatement of accumulated depreciation of $24,000 Understatement of accumulated depreciation of $40,000 Understatement of depreciation expense of $24,000 Understatement of net income of $24,000

Understatement of accumulated depreciation of $40,000 Failure to report depreciation expense would understate depreciation expense and accumulated depreciation by the amount of the depreciation expense not reported. If no other errors were made, income would be overstated by the net of tax amount ($24,000).

Vadis Co. sells appliances that include a 3-year warranty. Service calls under the warranty are performed by an independent mechanic under a contract with Vadis. Based on experience, warranty costs are estimated at $30 for each machine sold. When should Vadis recognize these warranty costs? Evenly over the life of the warranty When the service calls are performed When payments are made to the mechanic When the machines are sold

When the machines are sold Proper matching of revenues and expenses requires that if it is probable customers will make claims under warranties relating to goods that have been sold and a reasonable estimate of the costs can be made, the accrual method must be used. Therefore, Vadis Co. should recognize its warranty costs when the machines are sold.

Subsequent events are events or transactions that occur after the balance sheet date but before financial statements are issued or are available to be issued. When are financial statements considered available to be issued? When they are complete in a form and format that complies with GAAP When all approvals necessary for issuance have been obtained When they are complete in a form and format that complies with GAAP and all approvals necessary for issuance have been obtained When either they are complete in a form and format that complies with GAAP or all approvals necessary for issuance have been obtained

When they are complete in a form and format that complies with GAAP and all approvals necessary for issuance have been obtained Subsequent events are events or transactions that occur after the balance sheet date but before financial statements are issued or are available to be issued. Financial statements are considered available to be issued when they are complete in a form and format that complies with GAAP and all approvals necessary for issuance have been obtained, for example, from management, the board of directors, and/or significant shareholders. An entity that has a current expectation of widely distributing its financial statements to its shareholders and other financial statement users must evaluate subsequent events through the date that the financial statements are issued. All other entities must evaluate subsequent events through the date that the financial statements are available to be issued. (FASB ASC 855-10-20)

Green Co. incurred leasehold improvement costs for its leased property. The estimated useful life of the improvements was 15 years. The remaining term of the nonrenewable lease was 20 years. These costs should be: expensed as incurred. capitalized and depreciated over 20 years. capitalized and expensed in the year in which the lease expires. capitalized and depreciated over 15 years.

capitalized and depreciated over 15 years. The leasehold improvements will benefit a number of years and therefore must be capitalized and depreciated over the periods benefited. The improvements are expected to have a useful life of 15 years and should be depreciated over that period since it is shorter than the remaining lease term.

Davidson Corporation is contending a tax judgment with the IRS. Davidson's management can estimate the amount of loss that will occur if the judgment goes against Davidson. Davidson's legal staff judges the probability of loss to be probable. In this situation, a loss contingency should be: neither accrued as a liability nor disclosed. accrued as a liability but not disclosed. disclosed and accrued as a liability. disclosed, but not accrued as a liability.

disclosed and accrued as a liability. FASB ASC 450-20-25-2 requires accrual of a loss contingency when a loss is probable and the amount of such loss can be reasonably estimated. When a loss is not accrued because of the absence of one or both of these conditions, but a loss is reasonably possible, a loss contingency should be disclosed but not accrued. When the probability of a loss is remote, loss contingency is neither accrued nor disclosed.

Lessees initially measure variable lease payments based on an index or a rate; they must prepare their financial statements in accordance with: IFRS. U.S. GAAP. either IFRS or U.S. GAAP. neither IFRS nor U.S. GAAP.

either IFRS or U.S. GAAP. Under both IFRS (International Financial Reporting Standards) and U.S. GAAP, lessees initially measure variable lease payments based on an index (e.g., Consumer Price Index) or a rate (e.g., LIBOR (London Interbank Offered Rate)).

Some events provide evidence regarding conditions that did not exist on the balance sheet date, but arose subsequently and do not require an adjustment of the balance sheet. Assuming that the item is material, an example of a subsequent event that requires adjustment is: sale of bonds. loss from inventory fire. stock splits. loss on account receivable resulting from customer's bankruptcy.

loss on account receivable resulting from customer's bankruptcy. The loss on account receivable resulting from a customer's bankruptcy relates to an account that existed on the balance sheet date and an adjustment is needed. FASB ASC 855-10 provides guidance as to subsequent events that require recognition: "An entity shall recognize in the financial statements the effects of all subsequent events that provide additional evidence about conditions that existed at the date of the balance sheet, including the estimates inherent in the process of preparing financial statements." (FASB ASC 855-10-25-1) "The following are examples of recognized subsequent events: "If the events that gave rise to litigation had taken place before the balance sheet date and that litigation is settled, after the balance sheet date but before the financial statements are issued or are available to be issued, for an amount different from the liability recorded in the accounts, then the settlement amount should be considered in estimating the amount of liability recognized in the financial statements at the balance sheet date. "Subsequent events affecting the realization of assets, such as receivables and inventories or the settlement of estimated liabilities, should be recognized in the financial statements when those events represent the culmination of conditions that existed over a relatively long period of time. For example, a loss on an uncollectible trade account receivable as a result of a customer's deteriorating financial condition leading to bankruptcy after the balance sheet date but before the financial statements are issued or are available to be issued ordinarily will be indicative of conditions existing at the balance sheet date. Thus, the effects of the customer's bankruptcy filing shall be considered in determining the amount of uncollectible trade accounts receivable recognized in the financial statements at the balance sheet date." (FASB ASC 855-10-55-1)

A lessee measures their right-of-use asset at the present value of lease payments, less any guaranteed residual value; they must prepare their financial statements in accordance with: IFRS. U.S. GAAP. either IFRS or U.S. GAAP. neither IFRS nor U.S. GAAP.

neither IFRS nor U.S. GAAP. Lessees measure the right-of-use assets under both IFRS (International Financial Reporting Standards) and U.S. GAAP. Right-of-use (ROU) assets under both start with the lease liability measurement, which adds guaranteed residual values to the liability and ROU, rather than subtracting it.

Daniel Company is embroiled in a lawsuit with an individual investor. If the probability of loss from the lawsuit is remote, a loss contingency should be: neither accrued as a liability nor disclosed. accrued as a liability but not disclosed. disclosed and accrued as a liability. disclosed but not accrued as a liability.

neither accrued as a liability nor disclosed. FASB ASC 450-20-25-2 requires that when the probability of a loss is remote, the loss contingency is neither accrued nor disclosed. When a loss is probable and the amount of such loss can be reasonably estimated, the loss contingency should be accrued. When a loss is reasonably possible, a loss contingency should be disclosed but not accrued.

A company performing its long-lived asset impairment testing is reviewing the fair value of equipment. Each of the following valuation techniques may be appropriate for measuring the fair value of the equipment, except the: market approach. income approach. cost approach. net realizable value approach.

net realizable value approach. The net realizable value approach is not an acceptable fair valuation method. The determination of fair value may require the use of one or more valuation techniques. The valuation technique used should be consistent with the market approach, income approach, and/or cost approach, as appropriate. The market approach uses prices and other relevant information generated by market transactions involving identical or comparable assets or liabilities. The income approach converts future amounts (e.g., cash flows) to a single present amount (discounted) using methods such as present value techniques and option-pricing models. The cost approach is based on the amount that currently would be required to replace the service capacity of an asset (i.e., current replacement cost).

On January 1, 20X2, to better reflect the variable use of its only machine, Holly, Inc., elected to change its method of depreciation from the straight-line method to the units of production method. The original cost of the machine on January 2, 20X0, was $50,000, and its estimated life was 10 years. Holly estimates that the machine's remaining total life is 50,000 machine hours as of January 1, 20X2. Machine hours usage was 8,500 during 20X1 and 3,500 during 20X0. Holly's income tax rate is 30%. Holly should report the accounting change in its 20X2 financial statements as: a cumulative effect of a change in accounting principle of $1,400 in its income statement. an adjustment to beginning retained earnings of $1,400. an adjustment to beginning retained earnings of $2,000. prospectively.

prospectively. FASB ASC 250-10-45-5 requires that changes in depreciation methods be accounted for prospectively. Accordingly, the carrying amount (book value) of the machine at January 1, 20X2, should be depreciated over the remaining life of the machine. At January 1, 20X2, the carrying amount is $40,000 (i.e., $50,000 acquisition cost less $10,000 accumulated depreciation), which should be depreciated over the remaining 50,000 machine hours at a rate of $.80 per machine hour (i.e., $40,000 ÷ 50,000 hours = $.80). The depreciation amount to be recognized in 20X2 is the number of machine hours used in 20X2 × $.80.

A balance arising from the translation or remeasurement of a subsidiary's foreign currency financial statements is reported in the consolidated income statement when the subsidiary's functional currency is: neither the foreign currency nor the U.S. dollar. the U.S. dollar. the foreign currency. both the foreign currency and the U.S. dollar.

the U.S. dollar. The objective of translation or remeasurement is to report the subsidiary's income statement results in the U.S. parent's currency—which is the U.S. dollar.

The initial measurement of the right-of-use asset includes all of the following except: the initial measurement of the lease liability. any initial direct costs incurred by the lessee. any lease payment made at or before the lease commencement date, less any incentives received. the amount of any fee related to extending the initial lease term.

the amount of any fee related to extending the initial lease term. The right-of-use (ROU) asset is initially measured as the lease liability plus any lease payments made to the lessor at or before the commencement date, minus any lease incentives received, plus any initial direct costs incurred by the lessee. The lessee should include initial direct costs in its initial measurement of the ROU asset and the initial direct costs are deferred and amortized over the term of the lease on a straight-line basis for both finance leases and operating leases. The ROU asset does not include fees to extend the initial lease term.

Factors an entity should assess in determining whether the lessee has a significant incentive to exercise an option to extend the lease include all of the following except: the amount of lease payments in any optional period. the amount of contingent payments. costs relating to the termination of the lease. the amount of the fixed lease payments for the original lease period.

the amount of the fixed lease payments for the original lease period. The amount of the fixed lease payments for the original lease period would not be a factor in assessing whether to extend a lease as they would be a sunk cost and not relevant. The amount of lease payments in any optional period, the amount of contingent payments, and costs relating to the termination of the lease are all factors an entity should assess in determining whether the lessee has a significant incentive to exercise an option to extend the lease.

A. A. Corporation sells t-shirts displaying humorous sayings or pictures of popular artists. As such, they often have to deal with writedowns of inventories to present net realizable value that may only be able to be sold at reduced prices. A particular item, Shirt S, of which A. A. has 1,000 units, which originally cost $25, was written down to its net realizable value at the end of the prior year of $20, since it featured an artist that had not been producing any new material for some time. At the beginning of the new year, this artist introduced a new CD that was very popular, and the Shirt S item has also been selling at a price above what it had been selling for before. The present replacement cost to buy another unit of Shirt S now is $27. Since the item now has a net realizable value of $35, which is greater than its original cost, and since A. A. Corporation applies IFRS, it must carry the value of its inventory of Shirt S at: the original cost of $25. the net realizable value for this new year of $35. the net realizable value of the item last year of $20, since an inventory writedown cannot be reversed. the replacement cost of $27.

the original cost of $25. IFRS requires that inventories be valued at lower of cost or net realizable value. If net realizable value falls below cost, then inventory must be written down to net realizable value. If net realizable value increases back up above cost, then a recovery of the writedown must occur, and the inventory must be written back up to cost, which is now the lower of cost and net realizable value. Replacement cost is not used in this procedure.


Related study sets

Module 18: Musculoskeletal function

View Set

Peds - Chapter 31: Nursing Care During a Pediatric Emergency

View Set

Chemistry 1910 D-block elements: Metal properties

View Set

Language of Medicine Chapter 17- Abbreviations

View Set

11th grade semester 2 Spanish final

View Set

Research Methods Psych Ch 6: Research Strategies and Validity

View Set

AP BIO Cell Cycle and Signaling Test review

View Set

Clinical and Counseling Psychology

View Set